LSAT and Law School Admissions Forum

Get expert LSAT preparation and law school admissions advice from PowerScore Test Preparation.

 Administrator
PowerScore Staff
  • PowerScore Staff
  • Posts: 8950
  • Joined: Feb 02, 2011
|
#22632
Question #21: Strengthen. The correct answer choice is (B).

The stimulus describes a new gadget used to keep cattle in their pastures. The device uses GPS tracking and makes noise in the cow’s ears whenever the cow wanders off its pasture, steering it back home. Needless to say, coolness does not come cheap: apparently, outfitting all the cows with this device would be a far more expensive solution than, say, building fences to keep them contained. Nevertheless, the manufacturer is confident that the ranchers will shell out the cash anyway, and the stem asks us to support this prediction.

While technically a Strengthen question, it would be just as useful to think of it as a Resolve question: why would the ranchers purchase the device at its current price if there are cheaper ways of keeping the cattle in check? Maybe they are loaded and willing to splurge on cool gadgets? Possible, but highly unlikely. The LSAT assumes, for the most part, that people are rational actors. The correct answer choice must establish why purchasing the device would be a rational choice, not one driven by profligacy or stupidity.

Answer choice (A): At first glance, this may seem like an attractive answer. Recall, however, that you are supposed to strengthen the manufacturer’s prediction as it is: ranchers will purchase the device at its current price.

Answer choice (B): This is the correct answer choice, suggesting that ranchers will not need to outfit every single cow with the device. If only herd leaders get the gadget, the overall cost of using the device could be reduced to levels at or below the cost of building fences. Purchasing the device at its current price could be a rational choice after all, strengthening the prediction made by the device’s maker.

Answer choice (C): Just because the device causes no harm does not mean that the ranchers will necessarily buy it at its current price. They might, but only if the alternative means of keeping cattle in their pastures are somehow harmful. We do not know if they are, which is why answer choice (C) does not necessarily present a comparative advantage.

Answer choice (D): This is the Opposite answer, as it makes the manufacturer’s prediction even more baffling: if the device is just as effective as fences at keeping cattle in their pastures, then why not use the cheaper solution instead?

Answer choice (E): As with answer choice (A), this one does not support the exact prediction made in the stimulus. The manufacturer believes that ranchers will purchase the device at its current price, not at a discount.
 mrhansen
  • Posts: 2
  • Joined: Oct 24, 2019
|
#73168
This explanation makes total sense and matched my pre-phrase, but I eliminated the answer choice because the stimulus says, "outfitting all of the cattle in a heard with this device ... (emphasis on the all). Again, the explanation makes sense, but any tips on how to avoid this mistake on a future questions, when it seems to imply that all must be outfitted?
 James Finch
PowerScore Staff
  • PowerScore Staff
  • Posts: 943
  • Joined: Sep 06, 2017
|
#73198
Hi Mr. Hansen,

The big key here is to not make assumptions, and stick only to what is being stated as fact in the stimulus. When presented with a somewhat strange and tough-to-Prephrase answer choice like (B), don't immediately eliminate it, but rather take a second to look back over the stimulus and think about whether it actually affects a part of the argument in some way. It's almost impossible in the time provided to read certain stimuli once and understand that's going on in it from that one read. Going back to the stimulus after reading (B), we can see that it would have the effect of eliminating the premise about the cost of outfitting all the cattle, as only a few would need to be outfitted.

Hope this helps!
 VamosRafa19
  • Posts: 39
  • Joined: Nov 14, 2020
|
#81937
I eliminated B, because of the first part of the stimulus. If a cow strays outside it's pasture, it steers it back. My thought was that B assumed that there was a leader, which other cows would follow but if the cow strayed far enough from the herd then there wouldn't be a leader to follow. Is that too much of an assumption? I picked C because I treated as a resolve the paradox question and I figured that if it caused little stress to cattle then they'd buy it. Would C be better if it said device would cause less stress to cattle than other methods like fencing?
User avatar
 KelseyWoods
PowerScore Staff
  • PowerScore Staff
  • Posts: 1079
  • Joined: Jun 26, 2013
|
#81950
Hi VamosRafa19!

Yes, like the explanation above states, the problem with answer choice (C) is that it does not give the device a comparative advantage over the other methods. So it would be a stronger answer choice if it instead said that the device causes significantly less stress to the cattle than any other method of keeping the cattle in their pastures. Without this comparison, answer choice (C) is pretty useless because it doesn't give us any real reason to make up for the major price difference between the device and the other methods.

Careful with your thinking on answer choice (B). (B) states that "As they graze, cattle in a herd follow the lead of the same few members of the herd." If that's true, then you would just need to outfit the leaders with the device rather than the whole herd. The herd will follow the leaders. So if a leader strays away, the rest of the herd will follow, the device will steer the leader back to where it is supposed to be, and the rest of the herd will follow the leader back to where they are supposed to be. Other, non-leader cows follow the leaders, which means they are not going to stray away from the herd. So your scenario of a cow straying too far from the herd and then not having the leader wouldn't happen. Follower cows always stick with the herd. Leader cows have the device to keep them where they're supposed to be.

Hope this helps!

Best,
Kelsey
 VamosRafa19
  • Posts: 39
  • Joined: Nov 14, 2020
|
#82214
That makes sense, thanks Kelsey!

Get the most out of your LSAT Prep Plus subscription.

Analyze and track your performance with our Testing and Analytics Package.